How do you show |r_1-r_2| is rotationally invariant

  • Thread starter Physgeek64
  • Start date
  • Tags
    Invariant
In summary, the conversation revolves around showing that |r_1-r_2| is rotationally invariant, with the attempt at a solution involving transformations and a discussion about the correct transformation equation. The OP needs to find a linear transformation that satisfies certain conditions to prove the invariance of |r_1-r_2|.
  • #1
Physgeek64
247
11

Homework Statement


How do you show |r_1-r_2| is rotationally invariant

Homework Equations

The Attempt at a Solution


So i get that we need to show that it is invariant under the transformations

## r_1 \rightarrow r_1 + \epsilon (n \times r_1)##
## r_2 \rightarrow r_2 + \epsilon (n \times r_2)##

but that gives

## |r_1 + \epsilon (n \times r_1) -r_2 - \epsilon (n \times r_2)|##
and i can't see what this equals ##|r_1 -r_2|##

Many thanks
 
Physics news on Phys.org
  • #2
Physgeek64 said:

Homework Statement


How do you show |r_1-r_2| is rotationally invariant

Homework Equations

The Attempt at a Solution


So i get that we need to show that it is invariant under the transformations

## r_1 \rightarrow r_1 + \epsilon (n \times r_1)##
## r_2 \rightarrow r_2 + \epsilon (n \times r_2)##

but that gives

## |r_1 + \epsilon (n \times r_1) -r_2 - \epsilon (n \times r_2)|##
and i can't see what this equals ##|r_1 -r_2|##

Many thanks
What is n?
 
  • #3
How would you show that ##|r_1|## is invariant ?
 
  • #4
Physgeek64 said:

Homework Statement


How do you show |r_1-r_2| is rotationally invariant

Homework Equations

The Attempt at a Solution


So i get that we need to show that it is invariant under the transformations

## r_1 \rightarrow r_1 + \epsilon (n \times r_1)##
## r_2 \rightarrow r_2 + \epsilon (n \times r_2)##

but that gives

## |r_1 + \epsilon (n \times r_1) -r_2 - \epsilon (n \times r_2)|##
and i can't see what this equals ##|r_1 -r_2|##

Many thanks

So, you want to show that ##|r_1' - r_2'| = |r_1 - r_2|,## where ##r_1' = r_1 + \epsilon (n \times r_1)## and ##r_2' = r_2 + \epsilon (n \times r_2).## If ##s = r_1 - r_2## and ##s' = r_1' - r_2'##, we have that
$$s' = s + \epsilon (n \times s),$$
so
$$s'^2 = s' \cdot s' = s^2 + \epsilon (n \times s) \cdot s + \epsilon s \cdot (n \times s) + \epsilon^2 (n \times s) \cdot (n \times s).$$
Since ##n \times s \: \perp s##, the two middle terms (linear in ##\epsilon##) vanish. So, you need only worry about the ##\epsilon^2## term.

It is clear that the result you want is not true, but I think that is because you misrepresented how a rotation acts on a vector. In other words, ##r \to r + \epsilon (n \times r)## is incorrect.
 
  • #5
The reasoning by Ray shows that an infinitesmal rotation conserves ##\ \left | \vec r_1 - \vec r_2\right |##. I suppose that is sufficient to show it for a finite rotation too ?

Again: @Physgeek64 : how would you prove that ##\ \left | \vec r_1 \right |\ ## is invariant ?
 
  • #6
BvU said:
The reasoning by Ray shows that an infinitesmal rotation conserves ##\ \left | \vec r_1 - \vec r_2\right |##. I suppose that is sufficient to show it for a finite rotation too ?

Again: @Physgeek64 : how would you prove that ##\ \left | \vec r_1 \right |\ ## is invariant ?

OK: the transformation shown sort-of-works for small ##|\epsilon|##, but it fails for large values of ##|\epsilon|##.

The point is that the OP's transformation equation ##r \to r + \epsilon (n \times r)## is just not correct! It describes taking a vector r and then adding something perpendicular to it on to its end. That will give a right triangle with sides ##|r|## and ##|\epsilon (n \times r)|##, whose hypotenuse is not just ##|r|##, but something bigger---only very slightly bigger if ##|\epsilon|## is small.

The OP needs to find a linear transformation ##T = T(\epsilon)## with the property that for small ##|\epsilon|## we have ##Tr \approx r + \epsilon (n \times r)##.
 
Last edited:
  • #7
Ray Vickson said:
So, you want to show that ##|r_1' - r_2'| = |r_1 - r_2|,## where ##r_1' = r_1 + \epsilon (n \times r_1)## and ##r_2' = r_2 + \epsilon (n \times r_2).## If ##s = r_1 - r_2## and ##s' = r_1' - r_2'##, we have that
$$s' = s + \epsilon (n \times s),$$
so
$$s'^2 = s' \cdot s' = s^2 + \epsilon (n \times s) \cdot s + \epsilon s \cdot (n \times s) + \epsilon^2 (n \times s) \cdot (n \times s).$$
Since ##n \times s \: \perp s##, the two middle terms (linear in ##\epsilon##) vanish. So, you need only worry about the ##\epsilon^2## term.

It is clear that the result you want is not true, but I think that is because you misrepresented how a rotation acts on a vector. In other words, ##r \to r + \epsilon (n \times r)## is incorrect.

But is this not the transformation one makes in lagrangian mechanics to then say ##\frac{\partial L}{\partial \dot{q}} (n \times r) = constant ## by Noether's theorem?

Many thanks
 
  • #8
Physgeek64 said:
But is this not the transformation one makes in lagrangian mechanics to then say ##\frac{\partial L}{\partial \dot{q}} (n \times r) = constant ## by Noether's theorem?

Many thanks

You need to distinguish between the differential relationship and the "integrated" relationship. In other words, if ##\vec{r}(t) = T(t) \vec{r}_0## we have
$$\frac{d\vec{r}}{dt} = \vec{\omega} \times \vec{r},$$
but the relationship between ##\vec{r}(t)## and ##\vec{r}_0## is not quite that simple. It is certainly true that
$$ \vec{r}(t+\Delta t) \approx \vec{r}(t) + \Delta t (\vec{\omega} \times \vec{r}(t))$$
for small ##|\Delta t|##, but you cannot just let ##\Delta t## become "large" in this equation and still have a correct relationship.

Note added in edit: If all you want is to show that ##|\vec{r}(t)|## remains constant (at least for constant ##\vec{\omega}##) you do not actually need to find ##\vec{r}(t)## at all. Showing that ##|\vec{r}|## is constant is a lot easier than finding ##\vec{r}## itself.
 
Last edited:
  • Like
Likes BvU
  • #9
Ray Vickson said:
OK: the transformation shown sort-of-works for small ##|\epsilon|##, but it fails for large values of ##|\epsilon|##.

The point is that the OP's transformation equation ##r \to r + \epsilon (n \times r)## is just not correct! It describes taking a vector r and then adding something perpendicular to it on to its end. That will give a right triangle with sides ##|r|## and ##|\epsilon (n \times r)|##, whose hypotenuse is not just ##|r|##, but something bigger---only very slightly bigger if ##|\epsilon|## is small.

The OP needs to find a linear transformation ##T = T(\epsilon)## with the property that for small ##|\epsilon|## we have ##Tr \approx r + \epsilon (n \times r)##.
##\epsilon## is never intended to be large. The relationship given is the one of an infinitesimal rotation. It is sufficient to show it to first order in ##\epsilon## as it is a direct result that the derivative is zero and therefore it extends directly to the integrated version.
 
  • Like
Likes BvU
  • #10
To get back to @Physgeek64 : Is the reasoning by @Ray Vickson in post #4 clear to you ? And the completed proof (with the inclusion of what @Orodruin indicates in #9 ) ?

Just checking ... :wink:
 

Related to How do you show |r_1-r_2| is rotationally invariant

1. How do you prove that |r1-r2| is rotationally invariant?

The rotational invariance of |r1-r2| can be proven mathematically by showing that the magnitude and direction of the vector |r1-r2| remains unchanged under any rotation.

2. Can you explain the concept of rotational invariance in simpler terms?

Rotational invariance refers to the property of a vector or quantity that remains unchanged under rotation. In other words, rotating the system or coordinate axes does not affect the magnitude or direction of the vector or quantity.

3. What is the significance of proving |r1-r2| is rotationally invariant?

Proving the rotational invariance of |r1-r2| is important in various fields of science and engineering, such as mechanics, fluid dynamics, and electromagnetism. It allows us to simplify calculations and make accurate predictions about the behavior of physical systems.

4. What are the common methods used to show rotational invariance?

There are different approaches to proving rotational invariance, depending on the specific context and mathematical tools available. Some common methods include using vector algebra, linear transformations, and differential calculus.

5. Can you provide an example of a physical quantity that is rotationally invariant?

One example of a quantity that is rotationally invariant is the angular momentum of a rotating body. Regardless of the orientation or axis of rotation, the magnitude and direction of the angular momentum remain constant, making it a useful and powerful concept in physics.

Similar threads

  • Calculus and Beyond Homework Help
Replies
4
Views
1K
  • Calculus and Beyond Homework Help
Replies
6
Views
1K
  • Calculus and Beyond Homework Help
Replies
1
Views
2K
  • Calculus and Beyond Homework Help
Replies
10
Views
2K
  • Calculus and Beyond Homework Help
Replies
3
Views
612
  • Special and General Relativity
Replies
2
Views
987
  • Calculus and Beyond Homework Help
Replies
2
Views
1K
  • Engineering and Comp Sci Homework Help
Replies
29
Views
4K
  • Calculus and Beyond Homework Help
Replies
5
Views
1K
Replies
2
Views
1K
Back
Top